Had the party's economic theories been sound and had it succeeded in implementing its program, the inflation rate wou...

MACZ on December 23, 2019

Could you explain why E would be wrong compared to C?

I thought this would be C because, like in the stimulus, it gives a scenerio for what will happen if an increase/decrease happens then says the opposite happened (increase or decrease) and forms the opposite conclusion off that. C only says that it doesn't increase. If someone could clear this up for me that would be great, thanks!

Reply
Create a free account to read and take part in forum discussions.

Already have an account? log in

shunhe on December 28, 2019

Hi @MACZ,

This is a match the flaw question. Let's try to diagram what the stimulus tells us, and then find an answer choice that exhibits a parallel flaw.

PETS = party's economic theories were sound
PSIP = party had succeeded in implementing its program
IRLC = inflation rate would have lessened considerably

Premise:
PETS & PSIP - > ILRC (This comes from the first sentence of the stimulus)

Conclusion:
~IRLC - > ~PETS (This is the second sentence of the stimulus)

What exactly is the flaw here? Well, the conclusion attempts to take the contrapositive of the stimulus and use it to get ~PETS. But the contrapositive actually tells us the following:

~IRLC - > ~(PETS & PSIP)
Which can be rewritten as
~IRLC - > ~PETS v ~PSIP

And note that the contrapositive can be true if PETS & ~PSIP. In plan English, it's possible that the party's economic theories were sound, but that it hadn't succeeded in implementing its program. Thus, we need an answer that has two conditions in the sufficient condition (before the arrow), and then takes the contrapositive and mistakenly negates one when the other one could have been negated. This is what (C) tells us. When we diagram out (C), we get

SSS = succeeded in selling its subsidiaries
CPNP = used cash to purchase the new patent
SPD = stock price doubled in the last two years

Premise:
SSS & CPNP - > SPD

Conclusion:
~SPD - > ~SSS

And we can see that this is parallel to the flawed reasoning used in the stimulus, since it's possible that the company sold its subsidiaries but didn't use the cash to purchase the new patent.

(E) is wrong because it has another flaw. (E) doesn't have two parts in the sufficient condition, which we can diagram as

TV's new weather forecasting equipment worth investment - > Ratings improve & Accuracy of forecasts rise

Because (E) has two parts in the necessary condition (after the arrow), but not the sufficient one (before the arrow), we know that it can't be a parallel flaw and can rule it out. Hope this helps, and feel free to ask any follow-up questions if anything is still confusing.